Einheitliches PDF der Differenz von zwei rv


9

Ist es möglich, dass das PDF der Differenz zweier iid-Wohnmobile wie ein Rechteck aussieht (anstelle von beispielsweise dem Dreieck, das wir erhalten, wenn die Wohnmobile aus der gleichmäßigen Verteilung entnommen werden)?

dh ist es möglich, dass das PDF f von jk (für zwei iid rvs aus einer Verteilung) f (x) = 0,5 für alle -1 <x <1 hat?

Es gibt keine Einschränkungen für die Verteilung, aus der wir j und k nehmen, außer dass die min -1 und die max 1 ist.

Nach einigen Experimenten denke ich, dass dies unmöglich sein könnte.


Der Unterschied zwischen zwei gleichmäßigen Verteilungen ist eine dreieckige Verteilung. Wenn Sie also fragen, ob es möglich ist, einen Unterschied zwischen verschiedenen Gleichungen zu erhalten, lautet die Antwort nicht.
Tim

Gleiche Frage hier gestellt: math.stackexchange.com/questions/2048939/… bisher ohne Antworten!
kjetil b halvorsen

Es scheint in der Tat schwierig zu sein, Realisierungen außerhalb von zu vermeiden, wenn sowohl als auch eine Wahrscheinlichkeitsmasse nahe diesen Endpunkten haben. j k[1,1]jk
Christoph Hanck

2
Es ist nicht möglich. Meiner Erinnerung nach wird dies (in etwas anderer Form) bereits irgendwo vor Ort beantwortet. Ich werde sehen, ob ich es finden kann
Glen_b - Monica am

1
@Glen_b Möglicherweise rufen Sie stats.stackexchange.com/questions/125360/… zurück . Es ist nicht ganz ein Duplikat, aber, weil eine Differenz von iid Variablen, obwohl ausdrückbar als eine Summe eine Summe von Variablen mit nicht-identischen Verteilungen beinhalten könnte. Ich glaube, dass eine geringfügige Änderung meiner Lösung diesen Unterschied beheben wird. Die Lösung von Silverfish sieht so aus, als würde sie fast ohne Modifikation direkt angewendet, aber zuerst muss viel Fremdmaterial entfernt werden, um dies zu sehen. X + ( - Y ) ,XYX+(Y),
whuber

Antworten:


10

Satz: Es gibt keine Verteilung für die wenn .DistABU(1,1)A,BIID Dist


Beweis: Betrachten Sie zwei Zufallsvariablen mit der gemeinsamen charakteristischen Funktion . Bezeichnet ihre Differenz mit . Die charakteristische Funktion der Differenz ist:A,BIID DistφD=AB

φD(t)=E(exp(itD))=E(exp(it(AB)))=E(exp(itA))E(exp(itB))=φ(t)φ(t)=φ(t)φ(t)¯=|φ(t)|2.

(Die vierte Zeile dieser Arbeit folgt aus der Tatsache, dass die charakteristische Funktion hermitisch ist .) nun eine spezifische Form für :DU(1,1)φD

φD(t)=E(exp(itD))=Rexp(itr)fD(r)dr=1211exp(itr)dr=12[exp(itr)it]r=1r=1=12exp(it)exp(it)it=12(cos(t)+isin(t))(cos(t)+isin(t))it=12(cos(t)+isin(t))(cos(t)isin(t))it=122isin(t)it=sin(t)t=sinc(t).

wobei letzteres die (nicht normalisierte) sinc-Funktion ist . Um die Anforderungen für zu erfüllen , benötigen wir daher eine charakteristische Funktion mit der Quadratnorm, die gegeben ist durch:Distφ

|φ(t)|2=φD(t)=sinc(t).

Die linke Seite dieser Gleichung ist eine quadratische Norm und daher nicht negativ, während die rechte Seite eine Funktion ist, die an verschiedenen Stellen negativ ist. Daher gibt es keine Lösung für diese Gleichung, und daher gibt es keine charakteristische Funktion, die die Anforderungen für die Verteilung erfüllt. (Hutspitze an Fabian, weil er in einer verwandten Frage zu Mathematics.SE darauf hingewiesen hat .) Daher gibt es keine Verteilung mit den Anforderungen des Satzes.


3

Dies ist die Sichtweise eines Elektrotechnikers in dieser Angelegenheit, mit einem Standpunkt, der eher für dsp.SE als für stats.SE geeignet ist, aber egal.

Angenommen, und sind kontinuierliche Zufallsvariablen mit gemeinsamem PDF . Wenn dann bezeichnet , haben wir Die Cauchy-Schwarz-Ungleichung sagt uns, dass ein Maximum bei . Da tatsächlich die "Autokorrelations" -Funktion von die als "Signal" betrachtet wird, muss es tatsächlich ein eindeutiges Maximum bei und daher kann nicht wie gewünscht gleichmäßig verteilt werden. Alternativ, wennXYf(x)ZXY

fZ(z)=f(x)f(x+z) dx.
fZ(z)z=0fZfz=0Z fZ in der Tat eine einheitliche Dichte (denken Sie daran, dass es sich auch um eine Autokorrelationsfunktion handelt), wäre die "Leistungsspektraldichte" von (als Signal betrachtet) eine Sinusfunktion und somit keine nichtnegative Funktion, da alle Leistungsspektraldichten sein müssen . Ergo führt die Annahme, dass eine einheitliche Dichte ist, zu einem Widerspruch und daher muss die Annahme falsch sein.fZfZ

Die Behauptung, dass offensichtlich ungültig ist, wenn die gemeinsame Verteilung von und Atome enthält , da in einem solchen Fall die Verteilung von auch Atome enthält. Ich vermute, dass die Einschränkung, dass und ein PDF haben , aufgehoben und ein rein messungstheoretischer Beweis für den allgemeinen Fall erstellt werden kann, in dem und nicht unbedingt ein PDF haben (aber ihr Unterschied).X Y Z X Y X Y.fZU[1,1]XYZXYXY


1
Ein Teil davon scheint mir nicht richtig zu sein. Die charakteristische Funktion der -Verteilung ist die -Funktion, so dass eindeutig eine Art Fourier-Transformation zulässig ist. Ihre Logik scheint mir zu viel zu beweisen - sie scheint nicht nur zu beweisen, dass nicht einheitlich sein kann, sondern dass die gleichmäßige Verteilung überhaupt nicht existieren kann. Habe ich falsch verstanden? sinc Z.U(1,1)sincZ
Ben - Reinstate Monica

1
Ob die charakteristische Funktion von existiert oder nicht, ist nicht das Problem; es existiert. Das PDF von ist eine Autokorrelationsfunktion . Nun, die spektralen Leistungsdichte von jeder Autokorrelationsfunktion muss eine nicht negative Funktion sein. Die Annahme, dass zu einer Leistungsspektraldichte führt, die eine sinc-Funktion ist (die sowohl positive als auch negative Werte annimmt). Da dies keine gültige Leistungsspektraldichte ist (denken daran, dass eine Autokorrelationsfunktion ist), muss die Annahme, dass falsch ist.Z f ZU [ - 1 , 1 ] f Z f ZU [ - 1 , 1 ]U[1,1]ZfZU[1,1]fZfZU[1,1]
Dilip Sarwate
Durch die Nutzung unserer Website bestätigen Sie, dass Sie unsere Cookie-Richtlinie und Datenschutzrichtlinie gelesen und verstanden haben.
Licensed under cc by-sa 3.0 with attribution required.